Q24

 
rishisb
Thanks Received: 0
Jackie Chiles
Jackie Chiles
 
Posts: 29
Joined: February 28th, 2010
 
 
 

Q24

by rishisb Sun Aug 01, 2010 5:39 pm

Greetings, Atlas:

May I ask a hypothetical, inference question about this passage?

On question Q24, I thought that answer choice E was attractive _ until I re-read it and saw that it (slyly) blotches what the last paragraph says. In the end, I threw out E because Garber says that the high prices of blubs are not irrational; he does not say the same thing about their low prices.

But, I wonder what if answer choice E instead said, "Mackay mistakenly infers that the high prices of the bulbs were irrational."

First, if this were what choice E said, would it be a good/reasonable inference to make from the passage? Why/why not?

Second, if the answer is "yes", would the revised-choice E be the correct answer (as opposed to B) to question 24? If not, why would B still be a better answer than the revised E? Please explain your reasoning!

I ask because I would like to better understand what makes an answer choice a good --or: the "best"-- response to an inference question in RC.

With gratitude for your time and insights,

Rishi
 
giladedelman
Thanks Received: 833
LSAT Geek
 
Posts: 619
Joined: April 04th, 2010
 
 
 

Re: A hypothetical inference-answer-choice to Q24

by giladedelman Mon Aug 02, 2010 1:04 pm

A very interesting question, Rishi!

I think you're right about answer (E). Whether the high prices were irrational is indeed a major point of contention between the two economists; Mackay believes that they were, Garber that they were not. So we could infer that Garber would agree with that statement.

Now, as for your second question: on the LSAT, there will never be two valid answer choices for one question. In this case, if we grant that our revised version of (E) is a valid inference, there's no way to judge which one -- (E) or (B) -- is "more" valid.

Here's how you might think of the "best answer" concept: occasionally, you won't be 100 percent satisfied with the correct answer, but having definitively ruled out the other four -- because there will always be four incorrect answers -- you'll know that it's the "least bad" answer.

Does that make any sense? Let me know if you'd like to keep discussing this issue. Great question!
 
rishisb
Thanks Received: 0
Jackie Chiles
Jackie Chiles
 
Posts: 29
Joined: February 28th, 2010
 
 
 

Re: A hypothetical inference-answer-choice to Q24

by rishisb Mon Aug 02, 2010 2:59 pm

Hello, Gilad:

I really appreciate your help.

I wonder, however, why you’d say that it is correct to infer that Garber believes that Mackey thinks that the high-prices of tulips is "irrational?" To me, that is not necessarily true: Garber never says that Mackey (or anyone else) believes that the tulips were a speculative bubble if and only if he believes that the high-prices were "irrational."

Had he said that, I’d agree that we could say that my revised, hypothetical answer choice would be a reasonable inference. Could you please explain your reasoning?

Thank you!
 
giladedelman
Thanks Received: 833
LSAT Geek
 
Posts: 619
Joined: April 04th, 2010
 
 
 

Re: A hypothetical inference-answer-choice to Q24

by giladedelman Tue Aug 03, 2010 2:11 pm

So, Rishi, did you only ask the question to try to trick me? ;)

My reasoning is this: the passage tells us that Mackay considers the Dutch tulip market to be a speculative bubble, in which prices were driven up not by underlying asset value but by speculation -- eventually proven disastrously wrong -- about future prices.

I think this description of the workings of a speculative bubble maps pretty well onto the notion of "irrational" pricing, even though it's not mentioned explicitly in the account of Mackay's position. Another clue is that Garber, who "challenges Mackay's view," does so in part by arguing that the high prices were not irrational, precisely because they could be justified by long-term fundamental asset value. This certainly reinforces our (hypothetical) inference that Mackay believes the contrary.
 
rishisb
Thanks Received: 0
Jackie Chiles
Jackie Chiles
 
Posts: 29
Joined: February 28th, 2010
 
 
 

Re: A hypothetical inference-answer-choice to Q24

by rishisb Tue Aug 03, 2010 7:19 pm

Hello, Gilad:

How are ya?

Thank you, again, for responding to my post. And, no, sir: I didn't try to trick you. If anything, I'm learning to get un-tricked, and that it's a frustrating task, at times.

I should take a step-backwards at this point, because -- frankly-- I'm very unclear about a fundamental point of RC inference questions. Our Atlas RC book, as well as virtually every other such guide (e.g.: the one by Manhattan Gmat), says that the answer to an RC inference "must be true". From the way the phrase is put, it sounds as if an answer to an RC-inference question must be a choice that is necessarily, logically entailed by the passage's text. (What else could the phrase "must be true" mean?)

This type of rigid, strict entailment is seen, for example, in the claims "All men are mortal, Socrates is a man. Thus, Socrates is mortal." In this case case nothing can block the conclusion from being true.

Now, one of my problems with RC inference questions, is that the answer choices -- indeed: even the credited answers --hardly ever are as logically necessiated as the conclusion in Socrates case. For example, the claim that Mackey thought the high prices were "irrational" in the tulip passage is far more open to debate than the conclusion that Socrates is mortal above. The latter is logically entailed; the former is not.

My question, then, is this: What does it really mean that an answer to a RC-inference question "must be true?" If not logical entailment, then what?

If you could remove this confusion, I'd be grateful!

With thanks,

Rishi
User avatar
 
ManhattanPrepLSAT1
Thanks Received: 1909
Atticus Finch
Atticus Finch
 
Posts: 2851
Joined: October 07th, 2009
 
 
 

Re: A hypothetical inference-answer-choice to Q24

by ManhattanPrepLSAT1 Wed Aug 04, 2010 2:11 pm

Let me jump into this conversation with my two cents.

Gilad is right on with his assessment of correct answer choices on inference questions in the reading comprehension section. I think it's fair to say that many inference questions in the reading comprehension section don't have correct answers that are absolutely, positively, beyond a shadow of a doubt, provable by the text in the passage. Rather, they are the answer choices that are most supported by the text.

This is tricky, because in logical reasoning, when they ask you to find what can be inferred from the stimulus, we generally hold them to a much stricter level of certainty.

For reading comprehension, inference questions should be thought of as "most supported" rather than "must be true." While many times you'll be able to find direct support and will be certain that the answer choice is something that "must be true," that will not always be the case. Try to give them a bit more flexibility in RC as compared to LR...

I hope this helps!
 
rishisb
Thanks Received: 0
Jackie Chiles
Jackie Chiles
 
Posts: 29
Joined: February 28th, 2010
 
 
 

Re: A hypothetical inference-answer-choice to Q24

by rishisb Fri Aug 06, 2010 4:12 pm

Hi, Matt:

How are you?

Thank you for clarifying that the answer to an RC-inference question need not be logically 'entailed'. That is a very important point that gets blurred when people say that such an answer "must be true."

Anyway, that leads to my other question: Once we understand that an answer choice to an RC-inference question need not be logically entailed/true, how do we go about determining what inference answer choice is the best?

In the easy cases, I can tell, that the correct answer choice frequently re-phrases a point that's more or less explicitly made in the passage. But, as we talked about Tuesday night, I’m not sure what sorts of reasons make an answer choice the "best" in the harder cases such as PT 50, Q24 (riddled basin passage) [by the way: it’s ironic that Atlas’s solutions, posted on the forum, says that the answer is easy, if one understood the scale. I made the scale, but didn’t get it right: I thought two answers were equally good. ]

At bottom, the issue is not that I don’t understand the passage; e.g, I can make the scales. My issue is that I do not know the test authors’ reasoning that makes a particular answer choice the "best" one in RC inference questions.
User avatar
 
ManhattanPrepLSAT1
Thanks Received: 1909
Atticus Finch
Atticus Finch
 
Posts: 2851
Joined: October 07th, 2009
 
 
 

Re: A hypothetical inference-answer-choice to Q24

by ManhattanPrepLSAT1 Fri Aug 06, 2010 6:44 pm

You should only be able to find support for one of the answer choices. The others will mix up the terms, take a claim too far, be contradicted by the passage, or be completely unsupported.

We're going to get into a lesson about what are common characteristics of incorrect answer choices in class soon (lesson 6).

We should work on this in office hours though. Have you scheduled a lesson yet for this week? Maybe you could grab a few inference questions from RC that have been driving you nuts and we'll go through them line by line and isolate what makes each of the incorrect answers, well... incorrect :)
 
clarafok
Thanks Received: 5
Forum Guests
 
Posts: 98
Joined: December 27th, 2010
 
 
trophy
Most Thankful
trophy
First Responder
 

Re: Q24

by clarafok Sat Feb 05, 2011 3:03 am

hello,

can someone please explain why B is the answer? i don't really see where it says that the owners could at least recoup their original investments...is it from line 47 where it talks about the bulbs selling at reproduction cost? but did the owners buy the bulbs at a high price? or they just owned the original bulbs from the first place so as long as it sells at reproduction cost, they could recoup their original investments?

any help would be much appreciated!

thanks!
User avatar
 
ManhattanPrepLSAT1
Thanks Received: 1909
Atticus Finch
Atticus Finch
 
Posts: 2851
Joined: October 07th, 2009
 
 
 

Re: Q24

by ManhattanPrepLSAT1 Tue Feb 22, 2011 11:31 pm

You're real close to the location in the passage where answer choice (B) is supported. Right after the sentence about "reproduction costs" the passage states that, "this does not mean that the high prices of original bulbs are irrational, for earnings derivable from the millions of bulbs descendent from the original bulbs can be very high."

This tells us that many (which translates into "some") owners of high-priced bulbs could expect to recoup their original investments.

Hope that helps! Let me know if you still have a question on this one.
 
clare.ess
Thanks Received: 0
Vinny Gambini
Vinny Gambini
 
Posts: 9
Joined: March 15th, 2012
 
 
 

Re: Q24

by clare.ess Tue May 29, 2012 9:30 pm

I'm unclear on why C is incorrect. Isn't C supported by lines 35 (... this episode should not be descibed as a speculative bubble, for the increase + decline... can be explained in terms of the fundamentals) and line 48 (But this does not mean that the high prices of original bulbs are irrational...)
User avatar
 
ManhattanPrepLSAT1
Thanks Received: 1909
Atticus Finch
Atticus Finch
 
Posts: 2851
Joined: October 07th, 2009
 
 
 

Re: Q24

by ManhattanPrepLSAT1 Sat Jun 02, 2012 3:14 pm

Good question clare.ess!

Answer choice (C) reverses the logic of the lines you pointed to, which could be interpreted as saying that if the prices are rational, then there is not a speculative bubble. The author is not just equating the two ideas of rational prices and a speculative bubble but creating an argument that attempts to argue from rational prices to the absence of a speculative bubble.

It's really important on conditional statements (if... then...) that you determine what's the trigger and what's the outcome.

Does that answer your question?
 
alexg89
Thanks Received: 9
Jackie Chiles
Jackie Chiles
 
Posts: 39
Joined: July 24th, 2012
 
 
 

Re: Q24

by alexg89 Sat Sep 08, 2012 10:48 am

When I was taking this test I was debating between B and E and crossed off E because Mackay is not saying that the price drop is irrational he is saying it dropped due to market forces. The relevant area for E 44-49 shows Garber defining his views. He is showing that the price drop isn't irrational and then goes on to explain why his reasoning is different from Mackays. Lines 44-47 are where Garber states a point and then in lines 47-49 he immediately counters a possible objection and then further on explains in greater detail.
 
jayparkcom
Thanks Received: 1
Forum Guests
 
Posts: 17
Joined: October 24th, 2012
 
 
 

Re: Q24

by jayparkcom Sat Nov 17, 2012 5:22 pm

So I was stuck between B and E.

I can clearly see why B is the answer.

Anyway,

the "critical" part of 3rd passage is saying that, line 44 - 50, even if buying the original tulip at higher price isn't irrational because replicating them and reselling them would make up for the high price; and E is saying that if Mackay has something to say about it -in fact, make mistaken inferences about it.

To me, Mackay has nothing to say about it. Because both Garber and Mackay have different underlying ideas about how that high price - to -lower price tulip - phenomenon. And line 44 - 50 are just one of the supports that Garber presents for his argument?

But I feel like above posts are saying that Mackay does have something to say about it but not as described in E?
 
Nina
Thanks Received: 0
Atticus Finch
Atticus Finch
 
Posts: 103
Joined: October 15th, 2012
 
 
 

Re: Q24

by Nina Tue Apr 02, 2013 8:40 pm

why is E incorrect? is it because Mackay would think that the very high prices of original bulbs are irrational but not the "very low prices that the bulbs eventually sold"?

Thanks!
 
amil91
Thanks Received: 5
Elle Woods
Elle Woods
 
Posts: 59
Joined: August 02nd, 2013
 
 
 

Re: Q24

by amil91 Wed Nov 13, 2013 12:04 pm

Nina Wrote:why is E incorrect? is it because Mackay would think that the very high prices of original bulbs are irrational but not the "very low prices that the bulbs eventually sold"?

Thanks!

Yes. The passage actually doesn't say what McKay thinks is irrational, but in lines 44-53 explicitly talks about the high prices of the original bulbs being not irrational, and this is in the paragraph expressing Garber's view which contradicts McKay's, so you could infer that McKay believed the high original prices were irrational, but we really aren't told about the rationality of the low prices that the reproduction bulbs eventually settle to.
 
timsportschuetz
Thanks Received: 46
Elle Woods
Elle Woods
 
Posts: 95
Joined: June 30th, 2013
 
 
trophy
First Responder
 

Re: Q24

by timsportschuetz Fri Nov 29, 2013 10:09 pm

I think there is a much much more obvious reason for ruling out (C)! We should clearly note that Garber is very cautious in his evidence. He is not using categorical language but rather states evidence that "can" explain a phenomenon differently than Mackay. You should immediately notice the extreme categorical nature of answer choice (C)! Notice how Garber always qualifies his statements:
1) "the increase and eventual decline in bulb prices can be explained in terms..."
2) "...rise in the price of some original tulip bulbs could have resulted as tulips..."
3) "... eventual fall of tulip bulb prices need not indicate a speculative bubble."

Now, I challenge anyone to go back and look at answer choice (C) and attempt to confidently choose this answer. There is simply no way that (C) can be verified and/or proven based on the passage.